How to evaluate $lim_{n to infty} frac{(1+sqrt 2)^n+(1-sqrt 2)^n}{(1+sqrt 2)^n-(1-sqrt 2)^n}$?











up vote
2
down vote

favorite













Evaluate $$lim_{n to infty} frac{(1+sqrt 2)^n+(1-sqrt 2)^n}{(1+sqrt 2)^n-(1-sqrt 2)^n}.$$




I tried to expand using Newton's Binomial Theorem, but it didn't work.










share|cite|improve this question




















  • 8




    Divide throughout by $(1+sqrt{2})^n$ and observe that $left|dfrac{1-sqrt{2}}{1+sqrt{2}}right| < 1$.
    – Muralidharan
    Nov 15 at 16:22










  • @Muralidharan you might post your comment as an answer :)
    – Nosrati
    Nov 15 at 16:27










  • observe that $|1-sqrt{2}|<1$
    – Vasya
    Nov 15 at 16:51















up vote
2
down vote

favorite













Evaluate $$lim_{n to infty} frac{(1+sqrt 2)^n+(1-sqrt 2)^n}{(1+sqrt 2)^n-(1-sqrt 2)^n}.$$




I tried to expand using Newton's Binomial Theorem, but it didn't work.










share|cite|improve this question




















  • 8




    Divide throughout by $(1+sqrt{2})^n$ and observe that $left|dfrac{1-sqrt{2}}{1+sqrt{2}}right| < 1$.
    – Muralidharan
    Nov 15 at 16:22










  • @Muralidharan you might post your comment as an answer :)
    – Nosrati
    Nov 15 at 16:27










  • observe that $|1-sqrt{2}|<1$
    – Vasya
    Nov 15 at 16:51













up vote
2
down vote

favorite









up vote
2
down vote

favorite












Evaluate $$lim_{n to infty} frac{(1+sqrt 2)^n+(1-sqrt 2)^n}{(1+sqrt 2)^n-(1-sqrt 2)^n}.$$




I tried to expand using Newton's Binomial Theorem, but it didn't work.










share|cite|improve this question
















Evaluate $$lim_{n to infty} frac{(1+sqrt 2)^n+(1-sqrt 2)^n}{(1+sqrt 2)^n-(1-sqrt 2)^n}.$$




I tried to expand using Newton's Binomial Theorem, but it didn't work.







calculus sequences-and-series limits






share|cite|improve this question















share|cite|improve this question













share|cite|improve this question




share|cite|improve this question








edited Nov 15 at 16:41









amWhy

191k27223438




191k27223438










asked Nov 15 at 16:17









user69503

566




566








  • 8




    Divide throughout by $(1+sqrt{2})^n$ and observe that $left|dfrac{1-sqrt{2}}{1+sqrt{2}}right| < 1$.
    – Muralidharan
    Nov 15 at 16:22










  • @Muralidharan you might post your comment as an answer :)
    – Nosrati
    Nov 15 at 16:27










  • observe that $|1-sqrt{2}|<1$
    – Vasya
    Nov 15 at 16:51














  • 8




    Divide throughout by $(1+sqrt{2})^n$ and observe that $left|dfrac{1-sqrt{2}}{1+sqrt{2}}right| < 1$.
    – Muralidharan
    Nov 15 at 16:22










  • @Muralidharan you might post your comment as an answer :)
    – Nosrati
    Nov 15 at 16:27










  • observe that $|1-sqrt{2}|<1$
    – Vasya
    Nov 15 at 16:51








8




8




Divide throughout by $(1+sqrt{2})^n$ and observe that $left|dfrac{1-sqrt{2}}{1+sqrt{2}}right| < 1$.
– Muralidharan
Nov 15 at 16:22




Divide throughout by $(1+sqrt{2})^n$ and observe that $left|dfrac{1-sqrt{2}}{1+sqrt{2}}right| < 1$.
– Muralidharan
Nov 15 at 16:22












@Muralidharan you might post your comment as an answer :)
– Nosrati
Nov 15 at 16:27




@Muralidharan you might post your comment as an answer :)
– Nosrati
Nov 15 at 16:27












observe that $|1-sqrt{2}|<1$
– Vasya
Nov 15 at 16:51




observe that $|1-sqrt{2}|<1$
– Vasya
Nov 15 at 16:51










3 Answers
3






active

oldest

votes

















up vote
5
down vote













$lim_{n to infty} frac{(1+sqrt 2)^n+(1-sqrt 2)^n}{(1+sqrt 2)^n-(1-sqrt 2)^n}$
$= lim_{nto infty }frac{1+a^n}{1-a^n} $ such that
$a=frac{(1-2^{frac{1}{2}})}{(1+2^{frac{1}{2}})} $ , $|a|<1$. So
lim is equal to $frac{1-0}{1+0}=1$






share|cite|improve this answer



















  • 3




    Dadrahm.Maybe you could add |a| <1.
    – Peter Szilas
    Nov 15 at 16:49










  • Thank you . . .
    – Dadrahm
    Nov 15 at 17:43










  • Dahdram.A pleasure +.
    – Peter Szilas
    Nov 15 at 17:55


















up vote
1
down vote













Roughly, $|1-sqrt 2| lt 1$, so a high power of it will go to $0$. $1+sqrt 2 gt 1$, so a high power of it will be large and positive. We can ignore the two small terms and be left with the fixed ratio $1$. Depending on what theorems you have proved about limits that may be enough.






share|cite|improve this answer




























    up vote
    1
    down vote













    We have that




    • $|1-sqrt 2|<1 implies (1-sqrt 2)^n to 0$


    therefore



    $$frac{(1+sqrt 2)^n+(1-sqrt 2)^n}{(1+sqrt 2)^n-(1-sqrt 2)^n}sim frac{(1+sqrt 2)^n}{(1+sqrt 2)^n}=1$$



    or more rigoursly



    $$frac{(1+sqrt 2)^n+(1-sqrt 2)^n}{(1+sqrt 2)^n-(1-sqrt 2)^n}= frac{(1+sqrt 2)^n}{(1+sqrt 2)^n}frac{1+frac{(1-sqrt 2)^n}{(1+sqrt 2)^n}}{1-frac{(1-sqrt 2)^n}{(1+sqrt 2)^n}}to frac{1+0}{1-0}$$






    share|cite|improve this answer





















      Your Answer





      StackExchange.ifUsing("editor", function () {
      return StackExchange.using("mathjaxEditing", function () {
      StackExchange.MarkdownEditor.creationCallbacks.add(function (editor, postfix) {
      StackExchange.mathjaxEditing.prepareWmdForMathJax(editor, postfix, [["$", "$"], ["\\(","\\)"]]);
      });
      });
      }, "mathjax-editing");

      StackExchange.ready(function() {
      var channelOptions = {
      tags: "".split(" "),
      id: "69"
      };
      initTagRenderer("".split(" "), "".split(" "), channelOptions);

      StackExchange.using("externalEditor", function() {
      // Have to fire editor after snippets, if snippets enabled
      if (StackExchange.settings.snippets.snippetsEnabled) {
      StackExchange.using("snippets", function() {
      createEditor();
      });
      }
      else {
      createEditor();
      }
      });

      function createEditor() {
      StackExchange.prepareEditor({
      heartbeatType: 'answer',
      convertImagesToLinks: true,
      noModals: true,
      showLowRepImageUploadWarning: true,
      reputationToPostImages: 10,
      bindNavPrevention: true,
      postfix: "",
      imageUploader: {
      brandingHtml: "Powered by u003ca class="icon-imgur-white" href="https://imgur.com/"u003eu003c/au003e",
      contentPolicyHtml: "User contributions licensed under u003ca href="https://creativecommons.org/licenses/by-sa/3.0/"u003ecc by-sa 3.0 with attribution requiredu003c/au003e u003ca href="https://stackoverflow.com/legal/content-policy"u003e(content policy)u003c/au003e",
      allowUrls: true
      },
      noCode: true, onDemand: true,
      discardSelector: ".discard-answer"
      ,immediatelyShowMarkdownHelp:true
      });


      }
      });














       

      draft saved


      draft discarded


















      StackExchange.ready(
      function () {
      StackExchange.openid.initPostLogin('.new-post-login', 'https%3a%2f%2fmath.stackexchange.com%2fquestions%2f2999898%2fhow-to-evaluate-lim-n-to-infty-frac1-sqrt-2n1-sqrt-2n1-sqrt%23new-answer', 'question_page');
      }
      );

      Post as a guest















      Required, but never shown

























      3 Answers
      3






      active

      oldest

      votes








      3 Answers
      3






      active

      oldest

      votes









      active

      oldest

      votes






      active

      oldest

      votes








      up vote
      5
      down vote













      $lim_{n to infty} frac{(1+sqrt 2)^n+(1-sqrt 2)^n}{(1+sqrt 2)^n-(1-sqrt 2)^n}$
      $= lim_{nto infty }frac{1+a^n}{1-a^n} $ such that
      $a=frac{(1-2^{frac{1}{2}})}{(1+2^{frac{1}{2}})} $ , $|a|<1$. So
      lim is equal to $frac{1-0}{1+0}=1$






      share|cite|improve this answer



















      • 3




        Dadrahm.Maybe you could add |a| <1.
        – Peter Szilas
        Nov 15 at 16:49










      • Thank you . . .
        – Dadrahm
        Nov 15 at 17:43










      • Dahdram.A pleasure +.
        – Peter Szilas
        Nov 15 at 17:55















      up vote
      5
      down vote













      $lim_{n to infty} frac{(1+sqrt 2)^n+(1-sqrt 2)^n}{(1+sqrt 2)^n-(1-sqrt 2)^n}$
      $= lim_{nto infty }frac{1+a^n}{1-a^n} $ such that
      $a=frac{(1-2^{frac{1}{2}})}{(1+2^{frac{1}{2}})} $ , $|a|<1$. So
      lim is equal to $frac{1-0}{1+0}=1$






      share|cite|improve this answer



















      • 3




        Dadrahm.Maybe you could add |a| <1.
        – Peter Szilas
        Nov 15 at 16:49










      • Thank you . . .
        – Dadrahm
        Nov 15 at 17:43










      • Dahdram.A pleasure +.
        – Peter Szilas
        Nov 15 at 17:55













      up vote
      5
      down vote










      up vote
      5
      down vote









      $lim_{n to infty} frac{(1+sqrt 2)^n+(1-sqrt 2)^n}{(1+sqrt 2)^n-(1-sqrt 2)^n}$
      $= lim_{nto infty }frac{1+a^n}{1-a^n} $ such that
      $a=frac{(1-2^{frac{1}{2}})}{(1+2^{frac{1}{2}})} $ , $|a|<1$. So
      lim is equal to $frac{1-0}{1+0}=1$






      share|cite|improve this answer














      $lim_{n to infty} frac{(1+sqrt 2)^n+(1-sqrt 2)^n}{(1+sqrt 2)^n-(1-sqrt 2)^n}$
      $= lim_{nto infty }frac{1+a^n}{1-a^n} $ such that
      $a=frac{(1-2^{frac{1}{2}})}{(1+2^{frac{1}{2}})} $ , $|a|<1$. So
      lim is equal to $frac{1-0}{1+0}=1$







      share|cite|improve this answer














      share|cite|improve this answer



      share|cite|improve this answer








      edited Nov 15 at 17:42

























      answered Nov 15 at 16:42









      Dadrahm

      3448




      3448








      • 3




        Dadrahm.Maybe you could add |a| <1.
        – Peter Szilas
        Nov 15 at 16:49










      • Thank you . . .
        – Dadrahm
        Nov 15 at 17:43










      • Dahdram.A pleasure +.
        – Peter Szilas
        Nov 15 at 17:55














      • 3




        Dadrahm.Maybe you could add |a| <1.
        – Peter Szilas
        Nov 15 at 16:49










      • Thank you . . .
        – Dadrahm
        Nov 15 at 17:43










      • Dahdram.A pleasure +.
        – Peter Szilas
        Nov 15 at 17:55








      3




      3




      Dadrahm.Maybe you could add |a| <1.
      – Peter Szilas
      Nov 15 at 16:49




      Dadrahm.Maybe you could add |a| <1.
      – Peter Szilas
      Nov 15 at 16:49












      Thank you . . .
      – Dadrahm
      Nov 15 at 17:43




      Thank you . . .
      – Dadrahm
      Nov 15 at 17:43












      Dahdram.A pleasure +.
      – Peter Szilas
      Nov 15 at 17:55




      Dahdram.A pleasure +.
      – Peter Szilas
      Nov 15 at 17:55










      up vote
      1
      down vote













      Roughly, $|1-sqrt 2| lt 1$, so a high power of it will go to $0$. $1+sqrt 2 gt 1$, so a high power of it will be large and positive. We can ignore the two small terms and be left with the fixed ratio $1$. Depending on what theorems you have proved about limits that may be enough.






      share|cite|improve this answer

























        up vote
        1
        down vote













        Roughly, $|1-sqrt 2| lt 1$, so a high power of it will go to $0$. $1+sqrt 2 gt 1$, so a high power of it will be large and positive. We can ignore the two small terms and be left with the fixed ratio $1$. Depending on what theorems you have proved about limits that may be enough.






        share|cite|improve this answer























          up vote
          1
          down vote










          up vote
          1
          down vote









          Roughly, $|1-sqrt 2| lt 1$, so a high power of it will go to $0$. $1+sqrt 2 gt 1$, so a high power of it will be large and positive. We can ignore the two small terms and be left with the fixed ratio $1$. Depending on what theorems you have proved about limits that may be enough.






          share|cite|improve this answer












          Roughly, $|1-sqrt 2| lt 1$, so a high power of it will go to $0$. $1+sqrt 2 gt 1$, so a high power of it will be large and positive. We can ignore the two small terms and be left with the fixed ratio $1$. Depending on what theorems you have proved about limits that may be enough.







          share|cite|improve this answer












          share|cite|improve this answer



          share|cite|improve this answer










          answered Nov 15 at 16:54









          Ross Millikan

          287k23195364




          287k23195364






















              up vote
              1
              down vote













              We have that




              • $|1-sqrt 2|<1 implies (1-sqrt 2)^n to 0$


              therefore



              $$frac{(1+sqrt 2)^n+(1-sqrt 2)^n}{(1+sqrt 2)^n-(1-sqrt 2)^n}sim frac{(1+sqrt 2)^n}{(1+sqrt 2)^n}=1$$



              or more rigoursly



              $$frac{(1+sqrt 2)^n+(1-sqrt 2)^n}{(1+sqrt 2)^n-(1-sqrt 2)^n}= frac{(1+sqrt 2)^n}{(1+sqrt 2)^n}frac{1+frac{(1-sqrt 2)^n}{(1+sqrt 2)^n}}{1-frac{(1-sqrt 2)^n}{(1+sqrt 2)^n}}to frac{1+0}{1-0}$$






              share|cite|improve this answer

























                up vote
                1
                down vote













                We have that




                • $|1-sqrt 2|<1 implies (1-sqrt 2)^n to 0$


                therefore



                $$frac{(1+sqrt 2)^n+(1-sqrt 2)^n}{(1+sqrt 2)^n-(1-sqrt 2)^n}sim frac{(1+sqrt 2)^n}{(1+sqrt 2)^n}=1$$



                or more rigoursly



                $$frac{(1+sqrt 2)^n+(1-sqrt 2)^n}{(1+sqrt 2)^n-(1-sqrt 2)^n}= frac{(1+sqrt 2)^n}{(1+sqrt 2)^n}frac{1+frac{(1-sqrt 2)^n}{(1+sqrt 2)^n}}{1-frac{(1-sqrt 2)^n}{(1+sqrt 2)^n}}to frac{1+0}{1-0}$$






                share|cite|improve this answer























                  up vote
                  1
                  down vote










                  up vote
                  1
                  down vote









                  We have that




                  • $|1-sqrt 2|<1 implies (1-sqrt 2)^n to 0$


                  therefore



                  $$frac{(1+sqrt 2)^n+(1-sqrt 2)^n}{(1+sqrt 2)^n-(1-sqrt 2)^n}sim frac{(1+sqrt 2)^n}{(1+sqrt 2)^n}=1$$



                  or more rigoursly



                  $$frac{(1+sqrt 2)^n+(1-sqrt 2)^n}{(1+sqrt 2)^n-(1-sqrt 2)^n}= frac{(1+sqrt 2)^n}{(1+sqrt 2)^n}frac{1+frac{(1-sqrt 2)^n}{(1+sqrt 2)^n}}{1-frac{(1-sqrt 2)^n}{(1+sqrt 2)^n}}to frac{1+0}{1-0}$$






                  share|cite|improve this answer












                  We have that




                  • $|1-sqrt 2|<1 implies (1-sqrt 2)^n to 0$


                  therefore



                  $$frac{(1+sqrt 2)^n+(1-sqrt 2)^n}{(1+sqrt 2)^n-(1-sqrt 2)^n}sim frac{(1+sqrt 2)^n}{(1+sqrt 2)^n}=1$$



                  or more rigoursly



                  $$frac{(1+sqrt 2)^n+(1-sqrt 2)^n}{(1+sqrt 2)^n-(1-sqrt 2)^n}= frac{(1+sqrt 2)^n}{(1+sqrt 2)^n}frac{1+frac{(1-sqrt 2)^n}{(1+sqrt 2)^n}}{1-frac{(1-sqrt 2)^n}{(1+sqrt 2)^n}}to frac{1+0}{1-0}$$







                  share|cite|improve this answer












                  share|cite|improve this answer



                  share|cite|improve this answer










                  answered Nov 15 at 17:17









                  gimusi

                  87.7k74393




                  87.7k74393






























                       

                      draft saved


                      draft discarded



















































                       


                      draft saved


                      draft discarded














                      StackExchange.ready(
                      function () {
                      StackExchange.openid.initPostLogin('.new-post-login', 'https%3a%2f%2fmath.stackexchange.com%2fquestions%2f2999898%2fhow-to-evaluate-lim-n-to-infty-frac1-sqrt-2n1-sqrt-2n1-sqrt%23new-answer', 'question_page');
                      }
                      );

                      Post as a guest















                      Required, but never shown





















































                      Required, but never shown














                      Required, but never shown












                      Required, but never shown







                      Required, but never shown

































                      Required, but never shown














                      Required, but never shown












                      Required, but never shown







                      Required, but never shown







                      Popular posts from this blog

                      Plaza Victoria

                      In PowerPoint, is there a keyboard shortcut for bulleted / numbered list?

                      How to put 3 figures in Latex with 2 figures side by side and 1 below these side by side images but in...